You are on page 1of 32

Partial Solutions for Linear Algebra by Friedberg et al.

Chapter 1
John K. Nguyen December 7, 2011
1.1.8. In any vector space V , show that (a + b)(x + y ) = ax + ay + bx + by for any x, y V and any a, b F . Proof. Let x, y V and a, b F . Note that (a + b) F (it is a scalar). By (VS7 ), we have that (a + b)(x + y ) = (a + b)x + (a + b)y . By (VS8 ), (a + b)x = ax + bx. Likewise, (a + b)y = ay + by . Thus, (a + b)x + (a + b)y = ax + bx + ay + by . Finally, by (VS1 ), we have (a + b)(x + y ) = ax + ay + bx + by . 1.1.9. Prove Corollaries 1 and 2 of Theorem 1.1 and Theorem 1.2(c). a) Corollary 1. The vector 0 described in VS3 is unique. b) Corollary 2. The vector y described in VS4 is unique. c) Theorem 1.2 (c). a0 = 0 for each a F . Note that 0 denotes the zero vector. Proof of (a). Suppose 0 V such that x + 0 = x for all x V . By (VS3 ), x + 0 = x. It follows that x + 0 = x + 0. By (VS1 ), 0 + x = 0 + x. We apply the Cancellation Law for Vector Addition (Theorem 1.1 ) to obtain 0 = 0 as required. Proof of (b). Assume y V such that x + y = 0. That is, both y and y are additive inverses of x (VS4 ). Since x + y = 0 and x + y = 0, we have that x + y = x + y . By (VS1 ), we have y + x = y + x. We apply the Cancellation Law to get y = y as required. Proof of (c). Let a F . By (VS3 ), a0 = a(0 + 0). By (VS7 ), a0 = a0 + a0. By (VS1 ), 0 + a0 = a0 + a0. By the Cancellation Law, 0 = a0. Then, by (VS1 ), a0 = 0 as required. 1.2.21. Let V and W be vector spaces over a eld F . Let Z = {(v, w) : v V w W }. Prove that Z is a vector space over F with the operations (v1 , w1 ) + (v2 , w2 ) = (v1 + v2 , w1 + w2 ) and c(v1 , w1 ) = (cv1 , cw1 ). Proof. We will show that Z satisfy all of the eld axioms. We note that V and W are vector spaces. Let (v1 , x1 ), (v2 , x2 ), (v3 , x3 ) V and (y1 , w1 ), (y2 , w2 ), (y3 , w3 ) W. By (VS1 ), (v1 , x1 ) + (v2 , x2 ) = (v1 + v2 , x1 + x2 ) = (v2 , x2 ) + (v1 , x1 ) = (v2 + v1 , x2 + x1 ). Thus, v1 + v2 = v2 + v1 . Likewise, (y1 , w1 ) + (y2 , w2 ) = (y1 + y2 , w1 + w2 ) = (y2 , w2 ) + (y1 , w1 ) = (y2 + y1 , w2 + w1 ). Thus, w1 + w2 = w2 + w1 . Since v1 + v2 = v2 + v1 and w1 + w2 = w2 + w1 , by denition, (v1 , w1 ) + (v2 , w2 ) = (v1 + v2 , w1 + w2 ) = (v2 + v1 , w2 + w1 ) = (v2 , w2 ) + (v1 , w1 ). We have shown that (VS1 ) holds for Z . Next, by (VS2 ), ((v1 , x1 )+(v2 , x2 ))+(v3 , x3 ) = (v1 , x1 )+((v2 , x2 )+(v3 , x3 )) so (v1 + v2 )+ v3 = v1 +(v2 + v3 ). Similarly, ((y1 , w1 ) + (y2 , w2 )) + (y3 , w3 ) = (y1 , w1 ) + ((y2 , w2 ) + (y3 , w3 )) so (w1 + w2 ) + w3 = w1 + (w2 + w3 ). It follows that ((v1 + v2 ) + v3 , (w1 + w2 ) + w3 ) = (v1 + (v2 + v3 ), w1 + (w2 + w3 )). Thus, ((v1 , w1 ) + (v2 , w2 )) + (v3 , w3 ) = (v1 , w1 ) + ((v2 , w2 ) + (v3 , w3 )) where (v1 , w1 ), (v2 , w2 ), (v3 , w3 ) Z . We have shown that (VS2 ) holds for Z . By (VS3 ), there exists (0v1 , 0v2 ) V such that (xv , yv ) + (0v1 , 0v2 ) + 0v = (xv , yv ) for all (xv , yv ) V . Similarly, there exists (0w1 , 0w2 ) W such that (xw , yw ) + (0w1 , 0w2 ) = (xw , yw ) for all (xw , yw ) W . By denition, (0v1 , 0w1 ) Z . Thus, (v, w) + (0v1 , 0w1 ) = (v, w) and so (VS3 ) holds for Z . 1

By (VS4 ), there exists (v, x) V such that (v, x) + (v, x) = (0v1 , 0v2 ) for all (v, x) V . Likewise, there exists (y, w) V such that (y, w) + (y, w) = (0w1 , 0w2 ) for all (y, w) V . It follows that for all (v, w) Z , there exists (v, w) Z such that (v, w) + (v, w) = (0z1 , 0z2 ) where (0z1 , 0z2 ) Z so (VS4 ) holds for Z . For all (v, x) V , 1 (v, x) = (v, x) so 1 v = v (by (VS5 )). Also, for all (y, w) W , 1 (y, w) = (y, w) so 1 w = w. By denition (letting c = 1), 1 (v, w) = (1 v, 1 w) = (v, w) so (VS5 ) holds for Z . Let a, b F . By (VS6 ), for all (v, x) V , (ab)(v, x) = (a(b(v, x)). Likewise, for all (y, w) W , (ab)(y, w) = (a(b(y, w)). Accordingly, we have (ab)v = a(bv ) and (ab)w = a(bw). By denition, (ab)(v, w) = ((ab)v, (ab)w). It follows from above that ((ab)v, (ab)w) = (a(bv ), a(bw)) so (VS6 ) holds for Z . Let c F . By (VS7 ), c((v1 , x1 ) + (v2 , x2 )) = c(v1 , x1 ) + c(v2 , x2 ) = (cv1 , cx1 ) + (cv2 , cx2 ) which implies that c(v1 , v2 ) = (cv1 , cv2 ). Likewise, c((y1 , w1 ) + (y2 , w2 )) = c(y1 , w1 ) + c(y2 , w2 ) = (cy1 , cw1 ) + (cy2 , cw2 ) which implies c(w1 , w2 ) = (cw1 , cw2 ). Then, c((v1 , w1 )+(v2 , w2 )) = (c(v1 +v2 ), c(w1 +w2 )) = (cv1 +cv2 , cw1 +cw2 ) = c(v1 , w1 ) + c(v2 , w2 ) as required. So (VS7 ) holds for Z . For all a, b F , by (VS8 ), (a + b)(v, x) = ((a + b)v, (a + b)x) = (av + bv, ax + bx) where (v, x) V . This implies (a + b)v = av + bv . Similarly, (a + b)(y, w) = ((a + b)y, (a + b)w) = (ay + by, aw + bw) which implies (a + b)w = aw + bw. It follows that (a + b)(v, w) = ((a + b)v, (a + b)w) = (av + bv, aw + bw) where (v, w) Z . We have shown that (VS8 ) holds for Z . We have shown that Z is a vector space. 1.3.5. Prove that A + At is symmetric for any square matrix A. Proof. Let A Mnxn (F ). Note that a square matrix A is symmetric if At = A where At is the transpose of A. That is, we wish to show that (A + At )t = A + At . By properties of tranpose, (A + At )t = At + (At )t = At + A = A + At as required. 1.3.6. Prove that tr(aA + bB ) = atr(A) + btr(B ) for any A, B Mnxn (F ).
n n

Proof. Let A, B Mnxn (F ). By denition of trace, tr(A) =


i=0

Aii and tr(B ) =


i=0

Bii where i is the row


n

and column of the matrix (because the matrix is square). Thus, by denition, tr(aA + bB ) =
n n n n i=0 n

(aA + bB )ii = Bii . Finally,


i=0

(aAii + bBii ). Then, by property of summation, we have


i=0 i=0

aAii +
i=0

bBii = a
i=0

Aii + b

by denition of trace, atr(A) + btr(B ) as required. 1.3.18. Prove that a subset W of a vector space V is a subspace of V if and only if 0 W and ax + y W whenever a F and x, y W . Proof. () Suppose a subset W of a vector space V is a subspace with addition and scalar multiplication dened on V . By denition of subspace, we know that there exists 0 W . We also know that ax W (closed under scalar multiplication) whenever x W and a F . And since W is closed under addition, we have that ax + y W for all y W . () Suppose 0 W and ax + y W for any x, y W and a F . Since we already know that 0 W , we only need to show that W is closed under addition and scalar multiplication. Since a F , setting a = 1 F , we have that 1 x + y = x + y W so W is closed under addition. Since y W and 0 W , we let y = 0 W . This means that ax + 0 = ax W so W is closed under scalar multiplication. 1.3.19. Let W1 and W2 be subspaces of a vector space V . Prove that W1 W2 is a subspace of V if and only if W1 W2 or W2 W1 . 2

Proof. () Suppose W1 and W2 are subspaces of a vector space V and assume W1 W2 is a subspace. We wish to prove by contradiction so assume W1 W2 and W2 W1 . Since W1 W2 , there exists a W1 but a W2 . Also, since W2 W1 , there exists b W2 but b W1 . Since W1 W2 is a subspace, it must be that (a + b) W1 or (a + b) W2 . We have two cases. Case I: Assume (a + b) W1 . By the additive inverse property, a + b + (a) = b W1 which contradicts our assumption that b W1 . Case II: Assume (a + b) W2 . By the additive inverse property, a + b + (b) = a W2 which contradicts our assumption that a W2 . We have shown that if W1 W2 is a subspace, then W1 W2 or W2 W1 . () Suppose W1 W2 or W2 W1 . We have two cases. Case I: Assume W1 W2 . Then, by denition of union, W1 W2 = W2 which is a subspace of V . Case II: Assume W2 W1 . Then, by denition of union, W1 W2 = W1 which is a subspace of V . In either cases, we have that W1 W2 is a subspace if W1 W2 or W2 W1 . 1.3.23. Let W1 and W2 be subspaces of a vector space V . a) Prove that W1 + W2 is a subspace of V that contains both W1 and W2 . b) Prove that any subspace of V that contains both W1 and W2 must also contain W1 + W2 . Proof of (a). We dene W1 + W2 = {w1 + w2 |w1 W1 w2 W2 }. We rst show that W1 + W2 is a subspace. First, since 0 W1 and 0 W2 , we have that 0 = 0 + 0 W1 + W2 . Next, if w1 , w2 W1 + W2 , there exists x W1 and y W2 such that w1 = x + y . Similarly, there exists z W1 and t W2 such that w2 = z + t. Then, w1 + w2 = (x + y ) + (z + t) = (x + z ) + (y + t) W1 + W2 (since W1 and W2 are subspaces, x + z W1 and y + t W2 ) so W1 + W2 is closed under addition. Lastly, since W1 and W2 are subspaces, we have that qw1 W1 and qw2 W2 for any q F by denition. Thus, aw1 + aw2 = a(w1 + w2 ) W1 + W2 so W1 + W2 is closed under scalar multiplication. Thus, W1 + W2 is a subspace of V . We now show W1 W1 + W2 and W2 W1 + W2 . By denition of subspace, we know that every vector in W1 + W2 has an additive inverse which means that with any arbitrary vector we can always obtain 0 W1 + W2 . And since, 0 W1 and 0 W2 by denition of subspace, it follows that W1 W1 + W2 and W2 W1 + W2 . Proof of (b). Suppose W is a subspace of V . Assume W1 W and W2 W . We wish to prove that W1 + W2 W . Let u W1 + W2 . From part (a), we dene u = x + y where x W1 and y W2 . Since W1 W , x W . Similarly, since W2 W , y W . And since W is a subspace, we know that u = x + y W where x, y W . Since W was arbitrary, we have shown that any subspace of V that contains both W1 and W2 must also contain W1 + W2 . 1.3.30. Let W1 and W2 be subspaces of a vector space V . Prove that V is the direct sum of W1 and W2 if and only if each vector in V can be uniquely written as x1 + x2 where x1 W1 and x2 W2 . Proof. () Suppose W1 and W2 are subspaces of a vector space V . Assume that V is a direct sum of W1 and W2 (that is, V = W1 W2 ). Then, each vector in V can be written as x1 + x2 where x1 W1 and x2 W2 . We wish to show that x1 + x2 is unique so assume x1 + x2 = y1 + y2 where y1 W1 and y2 W2 . It follows that x1 y1 = y2 x2 . Because V = W1 W2 , we know that W1 W2 = {0}. Then, in knowing that W1 and W2 are disjoint and that x1 y1 W1 and y2 x2 W2 (W1 and W2 are subspaces of V so both are closed under addition), we have that x1 y1 = 0 and y2 x2 = 0. Thus, x1 = y1 and x2 = y2 so x1 + x2 is unique. () Let W1 and W2 be subspaces of a vector space V . Suppose each vector in V can be uniquely written as x1 + x2 where x1 W1 and x2 W2 . Then, it follows that W1 + W2 = V . Next, we let t be in the intersection of W1 and W2 (i.e. t W1 W2 ). Since W1 and W2 are subspaces, we know that 0 W1 and 3

0 W2 . Then, t = t + 0 where t W1 and 0 W2 . Also, t = 0 + t where 0 W1 and t W2 . It must follow that t = 0 so W1 W2 = {0} since t W1 W2 . Since W1 + W2 = V and W1 W2 = {0}, by denition of direct sum, we have V = W1 W2 as required. We conclude that V is the direct sum of W1 and W2 if and only if each vector in V can be uniquely written as x1 + x2 . 1.3.31. Let W be a subspace of a vector space V over a eld F . For any v V the set {v } + W = {v + w : w W } is called the coset of W containing v . It is customary to denote this coset by v + W rather than {v } + W . (a) Prove that v + W is a subspace of V if and only if v W . (b) Prove that v1 + W = v2 + W if and only if v1 v2 W . (c) Prove that the preceding operations are well dened; that is, show that if v1 + W = v ` 1 + W and v2 + W = v ` ` ` ` 2 + W , then (v1 + W ) + (v2 + W ) = (v 1 + W ) + (v 2 + W ) and a(v1 + W ) = a(v 1 + W) for all a F . (d) Prove that the set S is a vector space with the operations dened in (c).This vector space is called the quotient space of V modulo W and is denoted by V /W . Proof of (a). () Suppose W is a subspace of a vector space V over a eld F . Assume v + W is a subspace of V . Because v + W is a subspace, we have that v v + W since 0 v + W . Since v v + W and v + W is a subspace, it follows that v + v v + W (since v + W is closed under addition). We wish to prove by contradiction so suppose v W . Since v W , we have by denition that v + v v + W , a contradiction. Thus, v W as required. () Assume v W . We wish to prove that v + W is a subspace of V so we check the three parts of the definition of subspace. Since W is a subspace, it is closed under scalar multiplication and so v = (1)v W . Then, 0 = v + (v ) v + W by denition of v + W . Next, suppose x = v + a v + W and y = v + b v + W for a, b W . Then, x + y = (v + a) + (v + b) = v + (a + v + b). Since we had assumed v W and a, b W , it follows that (a + v + b) W . So by denition, x + y = v + (a + v + b) v + W where (a + v + b) W and v W so v + W is closed under addition. Finally, assume f F . Then, f x = f v + f a which we can rewrite as f x = v + (v ) + f v + f a. Since a W and W is a subspace, f a W . Likewise, since v W , f v W . And since we have already stated that v W , we know that (v ) + cv + ca W because W is closed under addition. Thus, f x v + W so v + W is closed under scalar multiplication. We have shown that 0 v + W , v + W is closed under addition and v + W is closed under scalar multiplication and so v + W is a subspace of V . We conclude that v + W is a subspace of V if and only if v W . Proof of (b). () Suppose W is a subspace of a vector space V over a eld F . Assume v1 + W = v2 + W . Suppose x v1 + W and y v1 + W . By denition of coset, we have that x = v1 + a where a W . Likewise, we have that y = v2 + b where b W . Since v1 + W = v2 + W , we have that x = y . That is, v1 + a = v2 + b which can be rewritten as v1 v2 = b a. Since a W , b W and W is a subspace, we have that b a W . Thus, since v1 v2 = b a, it follows that v1 v2 W as required. () Now suppose v1 v2 W . We wish to show that v1 + W = v2 + W . That is, v1 + W v2 + W and v2 + W v1 + W . () Let x v1 + W and y v2 + W . By denition of coset, we have that x = v1 + a where a W . Likewise, we have that y = v2 + b where b W . Since v1 v2 , a W , we set b = (v1 v2 ) + a. Then, y = v2 + (v1 v2 ) + a = v1 + a. Since x = v1 + a, we have that x = y so x v2 + W . Because x v1 + W and x v2 + W , v1 + W v2 + W . () Let x v1 + W and y v2 + W . Again, by denition of coset, we have that x = v1 + a where a W . Likewise, we have that y = v2 + b where b W . Since v1 v2 W and W is a subspace, we have that v2 v1 W (it is clear that v2 + (1)v1 W because W is closed under addition and scalar 4

multiplication). Now, since v2 v1 , b W , we set a = (v2 v1 ) + b. Then, x = v1 + (v2 v1 ) + b = v2 + b. Since y = v2 + b, x = y so y v1 + W . Since y v2 + W and y v1 + W , v2 + W v1 + W . Because v1 + W v2 + W and v2 + W v1 + W , we conclude that v1 + W = v2 + W as required. Proof of (c). Assume W is a subspace of a vector space V over a eld F . Suppose v1 + W = v ` 1 + W and v2 + W = v ` ` ` 2 + W . We wish to show (v1 + W ) + (v2 + W ) = (v 1 + W ) + (v 2 + W ). However, from part(b) we know this is equivalent to (v1 + v2 ) (v ` ` ` ` 1+v 2 ) W . Since v1 + W = v 1 + W and v2 + W = v 2 + W , by part (b), we have that v1 v ` ` 1 W and v2 v 2 W . Since W is a subspace, it is closed under addition so (v1 v ` ` ` ` 1 ) + ( v2 v 2 ) W . Rearranging terms, (v1 + v2 ) (v 1+v 2 ) W as required. Now we wish to prove that a(v1 + W ) = a(v ` 1 + W ) for all a F . However, we apply part (b) which means showing av1 av ` ` 1 W is equivalent. From above, we already have v1 v 1 W . Since W is a subspace, it is closed under scalar multiplication so we have that a(v1 v ` 1 ) W for all a F . By the distribution law, we conclude that av1 av ` 1 W as required. Proof of (d). To show that set S is a vector space with the operations dened in (c), we must verify the eld axioms. For (V S 1), let v1 + W S and v2 + W S where v1 , v2 V . Then, (v1 + W ) + (v2 + W ) = (v1 + v2 ) + W and (v2 + W ) + (v1 + W ) = (v2 + v1 ) + W by the denition in part (c). Since v1 , v2 V , by (V S 1), we know that v1 + v2 V and v2 + v1 V so v1 + v2 = v2 + v1 . Thus, (v1 + v2 )+ W = (v2 + v1 )+ W so (V S 1) holds. Now suppose v1 + W S , v2 + W S and v3 + W S where v1 , v2 , v3 V . Then, by the denition in part (c), ((v1 + W ) + (v2 + W )) + (v3 + W ) = ((v1 + v2 ) + W ) + (v3 + W ) = ((v1 + v2 ) + v3 ) + W = (v1 + (v2 + v3 )) + W = (v1 + W ) + ((v2 + v3 ) + W ) = (v1 + W ) + ((v2 + W ) + (v3 + W )) which shows that (V S 2) holds. Suppose 0 + W S (we can assume this since 0 V ). Also, let v + W S for some v V . Then, (0 + W ) + (v + W ) = (0 + v ) + W = v + W . Thus, (V S 3) holds. Next, let v + W S where v V . Let v + W S where, since V is closed under scalar multiplication, v V . Then, v + W + (v ) + W = (v v ) + W = 0 + W where 0 S . Therefore, (V S 4) holds. We now verify (V S 5). Let v + W S for some v V . Then, by the denition in part (c), 1(v + W ) = (1v ) + W = v + W as required so (V S 5) holds. Let a, b F and suppose v + W S for some v V . Then, (ab)(v + W ) = (ab)v + W = a(bv )+ W = a(bv + W ) so (V S 6) holds. For (V S 7), let v1 + W S and v2 + W S where v1 , v2 V . Choose a F . Then, a((v1 + W ) + (v2 + W )) = (a(v1 + v2 )) + W = (av1 + av2 ) + W = (av1 + W ) + (av2 + W ) = a(v1 + W ) + a(v2 + W ) as required. We have shown that (V S 7) holds. Finally, let v + W S where v V . Pick a, b F . Then, (a + b)(v + W ) = ((a + b)v ) + W = (av + bv ) + W = (av + W ) + (b + W ) = a(v + W ) + b(v + W ) so (V S 8) holds. In conclusion, since all of the eld axioms hold, S is a vector space. 1.4.13. Show that if S1 and S2 are subsets of a vector space V such that S1 S2 , then span(S1 ) span(S2 ). In particular, if S1 S2 and span(S1 ) = V , deduce that span(S2 ) = V . Proof. Suppose S1 and S2 are subsets of a vector space V and that S1 S2 . We wish to show that every element of span(S1 ) is contained in span(S2 ). Choose z span(S1 ). By denition of span, there exists x1 , x2 , ..., xn S1 such that z = a1 x1 + a2 x2 + ... + an xn where a1 , a2 , ..., an F . But, since S1 S2 ,

x1 , x2 , ..., xn S2 also which means that z span(S2 ) by denition of linear combination. Since z was arbitrary, we have that span(S1 ) span(S2 ). Suppose span(S1 ) = V . We now show that span(S2 ) = V . Since span(S1 ) = V , from above, V span(S2 ). However, we have that S2 V which means that span(S2 ) V . That is, x1 , x2 , ..., xn S2 is also contained in V so by denition, we know that span(S2 ) V . Since V span(S2 ) and span(S2 ) V , we have that span(S2 ) = V . 1.4.15. Let S1 and S2 be subsets of a vector space V . Prove that span(S1 S2 ) span(S1 ) span(S2 ). Give an example in which span(S1 S2 ) and span(S1 ) span(S2 ) are equal and one in which they are unequal. Proof. Suppose S1 and S2 be subsets of a vector space V . Let v span(S1 S2 ), Then, by denition of span, there exists x1 , x2 , ..., xn S1 S2 such that v = a1 x1 + a2 x2 + ... + an xn where a1 , a2 , ..., an F . Since x1 , x2 , ..., xn S1 S2 , by denition of set intersection x1 , x2 , ..., xn S1 and x1 , x2 , ..., xn S2 . From this, we know that v = a1 x1 + a2 x2 + ... + an xn span(S1 ) and v = a1 x1 + a2 x2 + ... + an xn span(S2 ). By denition of set intersection, since v span(S1 ) and v span(S2 ), we have that v span(S1 ) span(S2 ). Since v was arbitrary, we have shown that span(S1 S2 ) span(S1 ) span(S2 ). Examples. Suppose V = R2 , S1 = {(1, 3)} and S2 = {(2, 7)}. Then, S1 S2 = so span(S1 S2 ) = {0} by denition of subspace. Next, we have that span(S1 ) span(S2 ) = {0} because span(S1 ) = {(1a, 3a) : a R} and span(S2 ) = {(2b, 7b) : b R} have no elements in common. In this example, span(S1 S2 ) = span(S1 ) span(S2 ). Now consider V = R2 , S1 = {(8, 4)} and S2 = {(4, 2)}. Again, we have that S1 S2 = so span(S1 S2 ) = {0} by denition of subspace. However, since span(S1 ) = {(8a, 4a) : a R} and span(S2 ) = {(4b, 2b) : b R}, we know that span(S1 ) span(S2 ) = {0} (since span(S1 ) = span(S2 )). In this example, span(S1 S2 ) = span(S1 ) span(S2 ) 1.5.9. Let u and v be distinct vectors in a vector space V . Show that {u, v } is linearly dependent if and only if u or v is a multiple of the other. Proof. () Assume u and v are distinct vectors in a vector space V . Suppose {u, v } is linearly dependent. b Assume a, b F and that they are not all are zero. Then, au + bv = 0 which can be rewriten as u = a v. Since both a, b F cant be zero, we can assume a = 0. Thus, we see that v is a multiple of u. On the other hand, if we rewritten the equation to v = a b u and assume that b = 0, we have that u is a multiple of v . In conclusion, we have that u or v is a multiple of the other. () Now assume that u or v is a multiple of other. Then, we have that u = av or v = bu for a, b F . By the distributive law of logic, we have two cases. Case I: Suppose u = av . Then, 0 = av u = av + (1)u. Since 1 F , by denition of linear dependent, we have {u, v } is linear dependent. Case II: Suppose v = bu. Then, 0 = bu v = bu + (1)v . Since 1 F , by denition of linear dependent, we have {u, v } is linear dependent. In both cases, we have that {u, v } is linear dependent. In conclusion, we have shown that {u, v } is linearly dependent if and only if u or v is a multiple of the other 1.5.13. Let V be a vector space over a eld of characteristic not equal to two. (a) Let u and v be distinct vectors in V . Prove that {u, v } is linearly independent if and only if {u + v, u v } is linearly indepedent. (b) Let u,v , and w be distinct vectors in V . Prove that {u, v, w} is linearly independent if and only if {u + v, u + w, v + w} is linearly independent. 6

Proof of (a). () Suppose V is a vector space over a eld of characteristic not equal to two. Assume u and v are distinct vectors in V . Suppose {u, v } is linearly independent. We wish to show that {u + v, u v } is linearly independent which mean a(u + v ) + b(u v ) = au + av + bu bv = (a + b)u + (a b)v = 0 for some a, b F such that a = b = 0. Since we know that {u, v } is linearly independent, a + b = 0 and a b = 0. Since a + b = 0, we know that b = a which implies a b = a (a) = a + a = 2a = 0. Because the eld characteristic is not equal to two, we have a = 0. In a similar argument, since a b = 0, we know that a = b which implies that a + b = b + b = 2b = 0. And since the eld characteristic is not equal to two, b = 0. Thus, since a = b = 0, we have shown that {u + v, u v } is linearly independent. () Now assume {u + v, u v } is linearly independent. Then, a(u + v ) + b(u v ) = 0 for some a, b F and a = b = 0. Rearranging terms, au + av + bu + bv = (a + b)u + (a b)v = 0. We prove by contradiction so assume that {u, v } is linearly dependent. That is, cu + dv = 0 for some c, d F and that all cannot be zero. Since (a + b)u + (a b)v = 0 and cu + dv = 0, c = a + b and d = a b. Since {u, v } is linearly dependent, we have that c = 0 or d = 0. By the distributive law of logic, we have two cases. Case I: Assume that c = 0. Since c = a + b, we have a + b = 0 which implies {u + v, u v } is linearly dependent because a and b cannot both be 0 since a + b = 0 for a(u + v ) + b(u v ) = 0 (our assumption that c = 0 would not hold if they are both zero). Case II: Assume that d = 0. Since d = a b, we have a b = 0 which implies {u + v, u v } is linearly dependent because a and b cannot both be 0 since a b = 0 for a(u + v ) + b(u v ) = 0 (our assumption that c = 0 would not hold if they are both zero). In either case, we have that {u + v, u v } is linearly dependent which is a contradiction. Therefore, {u, v } is linearly independent. In conclusion, {u, v } is linearly independent if and only if {u + v, u v } is linearly indepedent Proof of (b). () Let u, v and w be distinct vectors in V . Assume V is a vector space over a eld of characteristic not equal to two. Suppose Assume that u, v, w is linearly independent which, by denition, means au + bv + cw = 0 for some a, b, c F and a = b = c = 0. We wish to prove that {u + v, u + w, v + w} is linearly independent. Then, d(u+v )+e(u+w)+f (v +w) = du+dv +eu+ew +f v +f w = (d+e)u+(d+f )v +(e+f )w = 0 for some d, e, f F and d = e = f = 0. Since u, v, w is linearly independent, d + e = 0, d + f = 0 and e + f = 0. We then solve for each variables. Since d + e = 0, d = e. Then, d + f = e + f = 0 which implies f = e. Since f = e, e + f = e + e = 2e = 0. Since the eld characteristic is not two, we have e = 0. In a similar argument, since f = e, e + f = f + f = 2f = 0 which implies f = 0 (since char(F ) = 2). Lastly, e + f = 0 implies that e = f . Since e = f , d + e = d f = 0 which implies d = f and so d + f = d + d = 2d = 0 which mean d = 0 (again, since char(F ) = 2). Thus, since d = f = e = 0, we have that {u + v, u + w, v + w} is linearly independent. () Now assume that {u + v, u + w, v + w} is linearly independent. Then, a(u + v ) + b(u + w) + c(v + w) = 0 where a, b, c F and a = b = c = 0. Rearranging terms we have a(u + v ) + b(u + w) + c(v + w) = au + av + bu + bw + cv + cw = (a + b)u + (a + c)v + (b + c)w = 0. We will prove by contradiction so assume {u, v, w} is linearly dependent. Then, du + ev + f w = 0 where d, e, f F and not all are zero. It follows that a + b = d, a + c = e and b + c = f because (a + b)u + (a + c)v + (b + c)w = 0 and du + ev + f w = 0. Next, because we assumed {u, v, w} is linearly dependent, we know that d, e, f F are not all zero so d = 0 or e = 0 or f = 0. By the distributive law of logic, we have three cases. Case I: Suppose d = 0. Since a + b = d, a + b = 0 which implies {u + v, u + w, v + w} is linearly dependent because a and b cannot both be equal to zero (recall that wed assumed a = b = c = 0). Case II: Suppose e = 0. Since a + c = e, a + c = 0 which implies {u + v, u + w, v + w} is linearly dependent because a and c cannot both be equal to zero.

Case III: Suppose f = 0. Since b + c = f , b + c = 0 which implies {u + v, u + w, v + w} is linearly dependent because b and c cannot both be equal to zero. Thus, we have that either a, b, or c is not equal to zero which means that {u + v, u + w, v + w} is linearly dependent so we have a contradiction. Therefore, {u, v, w} must be linearly independent. In conclusion, {u, v, w} is linearly independent if and only if {u + v, u + w, v + w} is linearly independent. 1.6.11. Let u and v be distinct vectors of a vector space V . Show that if {u, v } is a basis for V and a and b are nonzero scalars, then both {u + v, au} and {au, bv } are also bases for V . Proof. Let u and v be distinct vectors of a vector space V . Suppose {u, v } is a basis for V and a and b are nonzero scalars. We wish to show that {u + v, au} and {au, bv } are linearly independent and that they span V . We begin by proving for {u + v, au}. First, to show that {u + v, au} is linearly independent, we need to show that for x(u + v ) + y (au) = 0, x = y = 0 for scalars x, y F . Rearranging terms, we notice x(u + v ) + y (au) = xu + xv + yau = (x + ya)u + (x)v = 0. Now since {u, v } is linearly independent by denition of basis, it follows that wu + qv = 0 and w = q = 0 for scalars w, q F . It follows that x + ya = w = 0 and x = q = 0. Then, because x = 0, ya = 0. Since we know that a is a nonzero scalar, it is clear that y = 0. Thus, the only solution to x(u + v ) + y (au) = 0 is x = y = 0 so {u + v, au} is linearly independent. Second, to show that {u + v, au} span V , by denition of basis, we need to show for any p V , x ` (u + v ) + y `(au) = p where x `, y ` F . Again, rearranging terms, we have (` x+y `a)u + (` x)v = p. By denition of basis, {u, v } span V which implies that for any p V , x u + y v = p, where scalars x , y F . It follows that x `+y `a = x and y which is dened since a is nonzero. Thus, we have that x `=y . Since x `=y , we solve for y ` to obtain y `= x a y {u + v, au} span V (clearly, x `(u + v ) + y `(au) = y (u + v ) + x ( au ) = p ). Because { u + v, au } is linearly a independent and {u + v, au} span V , {u + v, au} is a basis for V . Next, we will prove that {au, bv } is a basis for V . First, we show that {au, bv } is linearly independent. That is, m(au) + n(bv ) = 0 is satisfy since m = n = 0. Rearranging terms, we obtain (ma)u + (nb)v = 0. Because {u, v } is linearly independent by denition of basis, it follows that hu + jv = 0 and h = j = 0 for scalars w, q F . This implies that ma = h = 0and nb = j = 0. Since we know that a and b are nonzero scalars, m = n = 0 so {au, bv } is linearly independent. Second, we will show that {au, bv } span V . That is, by denition of span, w (au) + q (bv ) = f for any f V . Again, rearranging terms, we have (wa )u + ( q b)v = f Since {u, v } span V , x u + y v = f which implies that wa =x and q b = y . Then, it follows that w = x a and y y q = b so w (au) + q (bv ) = x a (au) + b (bv ) = f . Clearly, {au, bv } span V . Thus, since {au, bv } is linearly independent and since it span V , by denition of basis, {au, bv } is a basis of V . In conclusion, we have shown that if {u, v } is a basis for V and a and b are nonzero scalars, then both {u + v, au} and {au, bv } are also bases for V . 1.6.22. Let W1 and W2 be subspaces of a nite-dimensional vector space V . Determine necessary and sucient conditions of W1 and W2 so that dim(W1 W2 ) = dim(W1 ). Proof. We believe that dim(W1 W2 ) = dim(W1 ) if and only if W1 W2 . We will prove this claim. () Let W1 and W2 be subspaces of a nite-dimensional vector space V . Suppose dim(W1 W2 ) = dim(W1 ). Let be a basis for W1 W2 . By denition of set intersection, we know that W1 W2 W1 so we can extend to be a basis for W1 . It follows that for any x W1 , x W1 W2 . This implies that W1 W1 W2 . Since W1 W2 W1 and W1 W1 W2 , we have that W1 W2 = W1 . Now, by denition of set intersection (specically, the property of subset intersection), W1 W2 = W1 implies that W1 W2 . () Now assume that W1 W2 . Let be a basis for W1 W2 . Since W1 W2 , by the denition of set intersection (property of subset intersection), it follows that W1 W2 = W1 . Since W1 W2 = W1 (which tells us that W1 W2 W1 ), can be extended to be a basis for W1 . This implies that dim(W1 W2 ) = dim(W1 ). We have shown that dim(W1 W2 ) = dim(W1 ) if and only if W1 W2 . 8

1.6.29. (a) Prove that if W1 and W2 are nite-dimensional subspaces of a vector space V , then the subspace W1 + W2 is nite-dimensional, and dim(W1 + W2 ) = dim(W1 ) + dim(W2 ) dim(W1 W2 ). (b) Let W1 and W2 be nite-dimensional subspaces of a vector space V , and let V = W1 + W2 . Deduce that V is the direct sum of W1 and W2 if and only if dim(V ) = dim(W1 ) + dim(W2 ). Proof of (a). Suppose W1 and W2 are nite-dimensional subspaces of a vector space V . Let be a basis for W1 W2 such that = {u1 , u2 , ..., uk } (since we know W1 and W2 are nite-dimensional). Since W1 W2 W1 , we extend to a basis of W1 so = {u1 , u2 , ..., uk , v1 , v2 , ..., vm }. Likewise, = {u1 , u2 , ..., uk , w1 , w2 , ..., wp }. We wish to show that = {u1 , u2 , ..., uk , v1 , v2 , ..., vm , w1 , w2 , ..., wp } which implies that W1 + W2 is nite-dimensional (as it contains k = m = p number of vectors) and that dim(W1 + W2 ) = k + m + p = k + m + p +(k k ) = (k + m)+(k + p) k = dim(W1 )+ dim(W2 ) dim(W1 W2 ). By denition of basis, we show that span W1 + W2 and that it is linearly independent. Note that span W1 and span W2 as they are both bases. That is, for any w1 W1 , a1 u1 + ... + ak uk + b1 v1 + ... + bm vm = w1 and, for any w2 W1 , a1 u1 + ... + ak uk + c1 w1 + ... + cp wp = w2 for scalars ak , bm , and cp respectively (Theorem 1.8 ). It is obvious that span W1 + W2 because a1 u1 + ... + ak uk + b1 v1 + ... + bm vm + c1 w1 + ... + cp wp = w1 + w2 for any w1 + w2 W1 + W2 (recall our given denition of W1 + W2 in problem 1.3.23 in Homework Set 1 ). To show that is linearly independent, we will prove by contradiction so assume that it is not ( is linearly dependent). Because is linearly independent (by denition of basis), we know that for a1 u1 + ... + ak uk + b1 v1 + ... + bm vm = 0, ak = bm = 0. Similarly, since is linearly indepdenent, for a1 u1 + ... + ak uk + c1 w1 + ... + cp wp = 0, we know that ak = cp = 0. But, we had assumed that is linearly dependent which implies that for a1 u1 + ... + ak uk + b1 v1 + ... + bm vm + c1 w1 + ... + cp wp = 0, we have that coecients ak , bm , and cp are not all zeros. Thus, we have a contradiction so must be linearly independent. By denition of basis, because span W1 + W2 and is linearly independent, we have that is a basis for W1 + W2 . Thus, it is nite-dimensional and dim(W1 + W2 ) = (k + m) + (k + p) k = dim(W1 ) + dim(W2 ) dim(W1 W2 ) as required. Proof of (b). Let W1 and W2 be nite-dimensional subspaces of a vector space V , and let V = W1 + W2 . () Suppose that V = W1 W2 . We wish to prove that dim(V ) = dim(W1 ) + dim(W2 ). Because V = W1 W2 , we know that W1 W2 = {0} which implies that dim(W1 W2 ) = 0 as W1 and W2 have no unique vectors in common (in fact, they dont have any vectors in common). So, noting that V = W1 + W2 , following from from part (a), we have dim(V ) = dim(W1 )+ dim(W2 ) dim(W1 W2 ) = dim(W1 )+ dim(W2 ) as required. () Suppose dim(V ) = dim(W1 ) + dim(W2 ). We wish to prove that V = W1 W2 . Since we know that V = W1 + W2 , we only need to show that W1 and W2 are disjoint (i.e., W1 W2 = {0}). From part (a), we know that dim(V ) = dim(W1 ) + dim(W2 ) dim(W1 W2 ). However, from our assumption, we have that dim(W1 W2 ) = 0 which means, by denition of dimension, there isnt any unique number of vectors in each basis for W1 W2 . This implies that W1 and W2 are disjoint. Therefore, since V = W1 + W2 and W1 W2 = {0}, by denition of direct sum, V = W1 W2 as required. We have shown that V is the direct sum of W1 and W2 if and only if dim(V ) = dim(W1 ) + dim(W2 ) 1.6.33. (a) Let W1 and W2 be subspaces of a vector space V such that V = W1 W2 . If 1 and 2 are bases for W1 and W2 , respectively, show that 1 2 = and 1 2 is a basis for V . (b) Conversely, let 1 and 2 be disjoint bases for subspaces W1 and W2 , respectively, of a vector space V . Prove that if 1 2 is a basis for V , then V = W1 W2 . Proof of (a). Let W1 and W2 be subspaces of a vector space V such that V = W1 W2 . Suppose 1 and 2 are bases for W1 and W2 respectively. Dene 1 = {u1 , u2 , ..., un } and 2 = {v1 , v2 , ..., vm }. 9

Then, by Since we span(1 ) lows that

denition of basis, 1 and 2 both are linearly independent and span W1 and W2 respectively. know that V = W1 W2 , we have that W1 W2 = . By Theorem 1.18, we know that = W1 and span(2 ) = W2 . Since span(1 ) = W1 , span(2 ) = W2 and W1 W2 = , it folspan(1 ) span(2 ) = which implies that 1 2 = .

Now, based on our denition of 1 and 2 , we dene 1 2 = {u1 , u2 , ..., un , v1 , v2 , ..., vm }. We rst show that 1 2 is linearly independent. However, we will prove by contradiction so assume that 1 2 is linearly dependent. That is, a1 u1 + ... + an un + b1 v1 + ... + bm vm = 0 where scalars a1 , ..., an and b1 , ..., bm are not all zero. Because, 1 is linearly independent, we have that a1 u1 + ... + an un = 0 where scalars a1 = a2 = ... = an = 0. Likewise, for 2 , b1 v1 + ... + bn vn = 0 where scalars b1 = b2 = ... = bm = 0. Thus, a contradiction so it is obvious that 1 2 is linearly independent. Second, we need to show that 1 2 span V . Note the Corollary on page 51 of the text which states if W is a subspace of a nite-dimensional vector space V , then any basis for W can be extended to a basis for V . Recall that W1 and W2 are subspaces of a vector space V . Therefore, we extend 1 and 2 to bases for V . Thus, since 1 and 2 span V (by denition of basis), it follows that 1 2 span V . Thus, since 1 2 is linearly independent and span V , by denition of basis, we have that 1 2 is a basis for V . In conclusion, we have proved that if 1 and 2 are bases for W1 and W2 , respectively, show that 1 2 = and 1 2 is a basis for V . Proof of (b). Let 1 and 2 be disjoint bases for subspaces W1 and W2 , respectively, of a vector space V (i.e., 1 2 = ). Assume 1 2 is a basis for V . We wish to prove that V = W1 W2 . That is, we need to show that W1 W2 = and V = W1 + W2 . Since 1 2 = , we have that span(1 2 ) = {0}. And since 1 is a basis for W1 , 1 span W1 . Likewise, we have that 2 span W2 . Then, by denition of span, span(1 ) = W1 and span(2 ) = W2 . Note from problem 1.4.15 of Homework Set 2 that span(W1 W2 ) span(W1 ) + span(W2 ). It follows that span(1 ) span(2 ) = W1 W2 = as 1 2 = . Now we wish to show that V = W1 + W2 . We know that span(1 ) = W1 and span(2 ) = W2 . Also recall that 1 2 is a basis for V . Then, by denition, span(1 2 ) = V . Suppose each vector in span(1 2 ) can be uniquely written as w1 + w2 where w1 span(1 ) and w2 span(2 ). Then, by problem 1.3.30 of Homework Set 2, it follows that V = W1 + W2 .

10

Partial Solutions for Linear Algebra by Friedberg et al. Chapter 2


John K. Nguyen December 7, 2011
2.1.14. Let V and W be vector spaces and T : V W be linear. (a) Prove that T is one-to-one if and only if T carries linearly independent subsets of V onto linearly independent subsets of W . (b) Suppose that T is one-to-one and that S is a subset of V . Prove that S is linearly independent if and only if T (S ) is linearly independent. (c) Suppose = {v1 , v2 , ..., vn } is a basis for V and T is one-to-one and onto. {T (v1 ), T (v2 ), ..., T (vn )} is a basis for W . Proof of (a). Suppose V and W be vector spaces and T : V W is linear. () Suppose T is one-to-one. Let S be a linearly independent subset of V . We wish to prove by contradiction so assume that T (S ) is linearly dependent. Since S is linearly independent, for a1 v1 + ... + an vn = 0, a1 = a2 = ... = an = 0. By denition of linear transformation from V to W , we have that if T is linear, then T (0) = 0. In this case, T (a1 v1 + ... + an vn ) = T (0) = 0. Note, by Theorem 2.5, this is equivalent to T being one-to-one as we had assumed. But we have that T (S ) is linearly dependent which implies that for vectors v1 , v2 , ..., vn S , a1 T (v1 ) + ... + an T (vn ) = 0 where scalars a1 , a2 , ..., an are not all zero. Clearly, this contradiction because this would mean a1 v1 + ... + an vn = 0 which implies T (a1 v1 + ... + an vn ) = T (0), contradicting the assumption that T is one-to-one. Therefore, T (S ) must be linearly independent. Thus, since S was arbitrary, T carries linearly independent subsets of V onto linearly independent subsets of W . () Suppose that T carries linearly independent subsets of V onto linearly independent subsets of W . We prove by contradiction so assume T is not one-to-one. Then, by denition of one-to-one, for some a, b V such that T (x) = T (y ). It follows that T (x) T (y ) = 0 which implies that x y N (T ) because T : V W is linear. Proof of (b). Let V and W be vector spaces and T : V W be linear. Suppose that T is one-to-one and that S is a subset of V . () Suppose that S is linearly independent. Then, by part (a), we have that T (S ) is linearly independent and so were done. () Now suppose that T (S ) is linearly independent. We wish to prove by contradiction so assume that S is linearly dependent. This implies that for v1 , v2 , ..., vn S , a1 v1 + ... + an vn = 0 where scalars a1 , a2 , ...an are not all zero. Since T : V W is linear, we have that a1 T (v1 ) + ... + an T (vn ) = 0, again, where scalars a1 , a2 , ..., an are not all zero. However, we had assumed that T (S ) is linearly independent so we have a contradiction. Thus, S is linearly independent as required. Proof of (c). Suppose = {v1 , v2 , ..., vn } is a basis for V and T is one-to-one and onto. We wish to show that T ( ) is linearly independent and span W . Since T is one-to-one and is linearly independent (by Prove that T ( ) =

denition of basis), by part (b), T ( ) is linearly independent. Next, since is a basis for V , by Theorem 2.2, R(T ) = span(T ( )). That is, span R(T ). Now, since T is onto, we know that R(T ) = W . So, since T ( ) span R(T ) and R(T ) = W , we have tha span W . Therefore, by denition of basis, since T ( ) is linearly independent and span W , T ( ) is a basis for W . 2.1.17. Let V and W be nite-dimensional vector spaces and T : V W be linear. (a) Prove that if dim(V ) < dim(W ), then T cannot be onto. (b) Prove that if dim(V ) > dim(W ), then T cannot be one-to-one. Proof of (a). Let V and W be nite-dimensional vector spaces and T : V W be linear. Assume dim(V ) < dim(W ). We will prove by contradiction so assume that T is onto. By the Dimension Theorem, since V is nite-dimensional then nullity (T ) + rank (T ) = dim(V ) which, by denition of nullity and rank, can be written equivalently as dim(N (T )) + dim(R(T )) = dim(V ). Since T is onto, by Theorem 2.5, rank (T ) = dim(V ) which is equivalent to dim(R(T )) = dim(V ) by denition of rank. This implies that dim(N (T ))+ dim(R(T )) = 0+ dim(R(T )) = dim(V ). However, since T is onto, we also know that R(T ) = W according to Theorem 2.5. Thus, we have that dim(W ) = dim(V ) which is a contradiction (based on transitivity). Therefore, T is not onto. Proof of (b). Let V and W be nite-dimensional vector spaces and T : V W be linear. Assume dim(V ) > dim(W ). We will prove by contradiction so assume that T is one-to-one. By the Dimension Theorem, since V is nite-dimensional then nullity (T ) + rank (T ) = dim(V ) which, by denition of nullity and rank, can be written equivalently as dim(N (T )) + dim(R(T )) = dim(V ). Since T is one-to-one, by Theorem 2.4, N (T ) = {0} so it follows that dim(N (T )) = 0. This implies that dim(R(T )) = dim(V ). Also, by by Theorem 2.5, since T is one-to-one we have that rank (T ) = dim(W ) which is equivalent to dim(R(T )) = dim(W ) by denition of rank. Therefore, since dim(R(T )) = dim(V ) and dim(R(T )) = dim(W ), it follows that dim(W ) = dim(V ) which is a contradiction to our assumption that dim(V ) > dim(W ) so T must not be one-to-one. 2.1.40. Let V be a vector space and W be a subspace of V . Dene the mapping : V V /W by (v ) = v +W for v V . (a) Prove that is a linear transformation from V to V /W and that N ( ) = W . (b) Suppose that V is nite-dimensional. Use (a) and the dimension theorem to derive a formula relating dim(V ), dim(W ), and dim(V /W ). Proof of (a). Suppose V is a vector space and W is a subspace of V . Dene the mapping : V V /W by (v ) = v + W for v V . We will rst prove that is a linear transformation from V to V /W . That is, by denition of linear transformation, we need to show that for all x, y V and c F , (x + y ) = (x) + (y ) and (cx) = c (x). Note, in Problem 1.3.31 of Homework Set 2, we have already showed that the coset operations are well dened. Let a, b V . By denition of , (a + b) = (a + b) + W . Then, by Problem 1.3.31 and our denition of , (a + b) + W = (a + W ) + (b + W ) = (a) + (b) as required. Next, by denition of , for any c F , (ca) = (ca) + W . According to the denition of coset in Problem 1.3.31, (ca) + W = c(a + W ) which, by denition of is c (a) as required. We now prove that N ( ) = W . By Theorem 2.1, N ( ) is a subspace of V . Recall from Problem 1.3.31 that we have proven that the quotient space of V modulo W (V/W) is a vector space so it is dened under all of the eld axioms. By denition of null space, N ( ) = {x V : (x) = 0}. Let n N ( ). Then, (n) = 0 but we know that 0 W since W is a subspace. Since n was arbitrary, we have that N ( ) W . Now let w W . Since W is closed under addition, w + 0 W . But, we also have that w + 0 N ( ) since (w + 0) = 0. Thus, we have that W N ( ). Since N ( ) W and W N ( ), N ( ) = W as required.

Proof of (b). We claim that dim(V ) + dim(V /W ) = dim(V ). From the Dimension Theorem, we know that dim(N ( )) + dim(R( )) = dim(V ). From part (a), we know that N ( ) = W which implies dim(W ) + dim(R( )) = dim(V ). Also, it follows that R( ) = V /W . Thus, dim(W ) + dim(V /W ) = dim(V ) 2.2.13. Let V and W be vector spaces, and let T and U be nonzero linear transformations from V into W . If R(T ) R(U ) = {0}, prove that {T, U } is a linearly independent subset of L(V, W ). Proof. Let V and W be vector spaces, and suppose T : V W and U : V W are nonzero. Assume R(T ) R(U ) = {0}. Since T is nonzero, there exists v1 V such that T (v1 ) = 0. Likewise for U , there exist v2 V such that U (v2 ) = 0. We wish to show by contradiction so assume aT + bU = T0 (note, T0 denotes the zero transformation) where scalars a, b F are not all zero. By the distributive law of logic, we have two cases.
b b Case I: Assume a = 0 and b = 0. Note, since a = 0, aT + bU = T0 can be rewritten as T = a U = U( a ) b by denition of linear transformation. Then, T (v1 ) = U ( a v1 ) R(U ) by denition of range. This implies that T (v1 ) R(T ) R(U ). But, we know that R(T ) R(U ) = {0} so, in consideration that T (v1 ) = 0, T (v1 ) R(T ) R(U ) contradicts the fact that T is nonzero. a a Case II: Assume a = 0 and b = 0. Note, since b = 0, aT + bU = T0 can be rewritten as U = b T = T ( b ). a Then, U (v2 ) = T ( b v2 ) R(T ) by denition of range. This implies that U (v2 ) R(T ) R(U ). But, we know that R(T ) R(U ) = {0} so, in consideration that U (v2 ) = 0, U (v2 ) R(T ) R(U ) contradicts the fact that U is nonzero.

In either case, we have a contradiction. Therefore, for aT + bU = T0 , we have that a = b = 0 which, by denition of linear independence, means that {T, U } is linearly independent. 2.2.15. Let V and W be vector spaces, and let S be a subset of V . Dene S 0 = {T L(V, W ) : T (x) = 0 for all x S }. Prove the following statement. (a) S 0 is a subspace of L(V, W ). 0 0 (b) If S1 and S2 are subsets of V and S1 S2 , then S2 S1 . 0 0 (c) If V1 and V2 are subspaces of V then (V1 + V2 ) = V1 V20 . Proof of (a). Let T, U S 0 and choose a F . By Theorem 1.3, we wish to show that T0 S 0 and addition and scalar multiplication is closed for S 0 . First, it is clear that for any x S , T0 (x) = 0 since T0 is the zero transformation. Next, by denition, (T + U )(x) = T (x) + U (x) = 0 + 0 = 0 so T + U S 0 which means that S 0 is closed under addition. Finally, we have that aT (x) = T (ax) = 0 by denition of linear transformation so aT (x) S 0 . Thus, we have that S 0 is a subspace of L(V, W ).
0 Proof of (b). Suppose S1 and S2 are subsets of V . Also assume S1 S2 . Let T S2 . By denition, we have that for all x S2 , T (x) = 0. However, since S1 S2 , we know that x S1 also. This implies that for 0 0 0 . all x S1 , T (x) = 0 so we have T S1 . By deition of subset, we can conclude that S2 S1

Proof of (c). Suppose V1 and V2 are subspaces of V . () Let T (V1 + V2 )0 . Then, by denition, for all a V1 + V2 , T (a) = 0. By the denition on page 22 of set addition, we have that a = v1 + v2 for all v1 V1 and v2 V2 so T (v1 + v2 ) = 0. Since V1 is a subspace, we know that 0 V1 . So, setting v1 = 0, we have that T (0 + v2 ) = T (v2 ) = 0 for all v2 V2 which implies that T V20 . With similar consideration, since V2 is a subspace, 0 V2 so we have that T (v1 + 0) = T (v1 ) = 0 for all v1 V1 . This implies that T V10 . Since T V20 and T V10 , it follows that T V10 V20 . Therefore, we have that (V1 + V2 )0 V10 V20 . () Now let T V10 V20 . By denition, we have that for all v1 V1 , T (v1 ) = 0. Likewise, for all v2 V2 , T (v2 ) = 0. Note that by denition of linear transformation, T (v1 ) + T (v2 ) = T (v1 + v2 ). So, T (v1 + v2 ) = 0 for all v1 + v2 V1 + V2 (by denition of subspace addition) which implies that T (V1 + V2 )0 . Thus, it follows that (V1 + V2 )0 V10 V20 . 3

Since (V1 + V2 )0 V10 V20 and (V1 + V2 )0 V10 V20 , we have that (V1 + V2 )0 = V10 V20 as required. 2.3.11. Let V be a vector space, and let T : V V be linear. Prove that T 2 = T0 if and only if R(T ) N (T ). Proof. Suppose V be a vector space and let T : V V be linear. () Suppose T 2 = T0 . Pick a R(T ). By denition of range, there exists v V such that T (v ) = a. Because T 2 = T0 and T (v ) = a, it follows that T 2 (v ) = T (T (v )) = T (a) = 0 which means a N (T ). Thus, we have that R(T ) N (T ). () Suppose R(T ) N (T ). Choose a R(T ). By denition of range, there exists v V such that T (v ) = a. However, since R(T ) N (T ), a N (T ). So, by denition of nullspace, we have that T (v ) = a = 0 which implies that T (a) = 0. Then, T 2 (v ) = T (T (v )) = T (a) = 0. Thus, we have that T 2 = T 0 . In conclusion, we have shown that T 2 = T0 if and only if R(T ) N (T ). 2.3.12. Let V , W , and Z be vector spaces, and let T : V W and U : W Z be linear. (a) Prove that if U T is one-to-one, then T is one-to-one. Must U also be one-to-one? (b) Prove that if U T is onto, then U is onto. Must T also be onto? (c) Prove that if U and T are one-to-one and onto, then U T is also. Proof of (a). Suppose V , W , and Z are vector spaces, and suppose that T : V W and U : W Z is linear. Assume U T is one-to-one. We wish to prove by contradiction so assume that T is not one-to-one. Then, by denition of linear transformation, there exists a, b V such that a = b and T (a) = T (b) which can be rewritten as T (a) T (b) = 0. By Theorem 2.10, it follows that U T (a) U T (b) = U (T (a) T (b)) = U (0) = 0. Since U T (a) U T (b) = 0, we have that U T (a) = U T (b) which is a contradiction since a = b and U T is assumed to be one-to-one. Therefore, T must be one-to-one. Proposition (a). We claim that U does not need to be one-to-one. Proof of Proposition (a). It is sucient to provide a counterexample. Let V = R, W = R2 and Z = R. Dene T : R R2 by T (r1 ) = (r1 , 0) and dene U : R2 R by U (r1 , r2 ) = r1 . Then, U T (r1 ) = U (T (r1 )) = U (r1 , 0) = r1 (since U T : R R). Thus, we have that T and U T is one-to-one. However, U is not one-to-one since an element of Z is mapped to by multiple elements of W . That is, we might have dierent r2 but r1 will always result from the transformation. For example, U (1, 1) = U (1, 2) = U (1, 3) = 1. Proof of (b). Suppose V , W , and Z are vector spaces, and suppose that T : V W and U : W Z is linear. Assume U T is onto. By Theorem 2.9, U T : V Z is linear. Since U T is onto, for all v V , U T (v ) = z where z Z . We wish to prove by contradiction so assume that U is not onto. Then, we know that not all vector in Z are mapped to (they do not have a corresponding vector from W ). That is, there exists z Z such that U (w) = z for all w W . Clearly this contradicts the assumption that U T is onto since all vectors in Z need to be mapped to by denition of onto. Thus, U is onto. Proposition (b). We claim that T does not need to be onto. Proof of Proposition (b). It is sucient to show a counterexample. Let V = R, W = R2 and Z = R. Dene T : R R2 by T (r1 ) = (r1 , 0) and dene U : R2 R by U (r1 , r2 ) = r1 . Clearly U T is onto as every element of Z will be mapped to (U is also onto as every element of Z is mapped to by how we dene U ). However, T is not onto since it cannot correspond to the second coordinate of W . That is, the second coordinate will always be zero by how we dened T so there will be elements of W that does not have anything mapped to. For example, (0, 3) W does not get mapped to. Proof of (c). Suppose V , W , and Z are vector spaces, and suppose that T : V W and U : W Z is linear. Assume U and T are one-to-one and onto. We rst show that U T is one-to-one. Suppose U T (x) = U T (y ). We wish to show that x = y by the 4

denition of one-to-one. Then, U (T (x)) = U (T (y )). Since U is one-to-one, we have that T (x) = T (y ). Since T is one-to-one, it follows that x = y as required. We now show that U T is onto. Let v V . Since T is onto, we have that T (v ) = w for some w W . Also, let z Z . Since U is onto, we have that U (w) = z . It follows that U T (v ) = U (T (v )) = U (w) = z which implies that U T is onto. In conclusion, if U and T are one-to-one and onto, then U T is also. 2.3.16. Let V be a nite-dimensional vector space, and let T : V V be linear. (a) If rank (T ) = rank (T 2 ), prove that R(T ) N (T ) = {0}. Deduce that V = R(T ) N (T ). (b) Prove that V = R(T k ) N (T k ) for some positive integer k . Proof of (a). Let V be a nite-dimensional vector space, and let T : V V be linear. Suppose rank (T ) = rank (T 2 ). Since V is nite-dimensional, by the Dimension Theorem, we have that rank (T ) + nullity (T ) = dim(V ) and rank (T 2 ) + nullity (T 2 ) = dim(V ) which implies that rank (T ) + nullity (T ) = rank (T 2 ) + nullity (T 2 ). Because rank (T ) = rank (T 2 ), it follows that nullity (T ) = nullity (T 2 ) which is equivalent to dim(N (T )) = dim(N (T 2 )). Clearly, we have that N (T ) is a subspace of N (T 2 ) because for all t N (T ), T 2 (t) = T (T (t)) = T (0) = 0 which implies t N (T 2 ) so N (T ) N (T 2 ). So, since dim(N (T )) = dim(N (T 2 )) and N (T ) is a subspace of N (T 2 ), we have that N (T ) = N (T 2 ). Choose x R(T ) and also let x N (T ). Since x R(T ), by denition of range, there exists a V such that T (a) = x. Also, since x N (T ), by denition of nullspace, T (x) = 0. Then, with consideration that T (a) = x, we have that T (x) = T (T (a)) = T 2 (a) = 0 so it follows that a N (T 2 ). But, since we have that N (T ) = N (T 2 ), a N (T ) also. And because a N (T ), by denition of nullspace, T (a) = 0 which, since we have that T (a) = x, means T (a) = x = 0. Since we had arbitrarily that x N (T ) and x R(T ), we can conclude that R(T ) N (T ) = {0}. Proposition (a). V = R(T ) N (T ) Proof of Proposition (a). By denition of direct sum, we wish to prove that R(T ) N (T ) = {0} and V = R(T ) + N (T ). From part (a), we already know that R(T ) N (T ) = {0}. Recall problem 1.6.29 from Homework Set 3. Clearly, R(T ) and N (T ) are nite-dimensional subspaces of vector space V so by part (a) of the exercise, we have that dim(R(T ) + N (T )) = dim(R(T )) + dim(N (T )) dim(R(T ) N (T )) and R(T ) + N (T ) is nite-dimensional. Since we have determined that R(T ) N (T ) = {0}, we have that dim(R(T ) + N (T )) = dim(R(T )) + dim(N (T )). By the Dimension Theorem, we know that dim(V ) = dim(R(T )) + dim(N (T )) so it follows that dim(V ) = dim(R(T ) + N (T )). Thus, since R(T ) + N (T ) and V is nite-dimensional and dim(V ) = dim(R(T ) + N (T )), it follows that V = R(T ) + N (T ) as required. In conclusion, since we have shown that R(T ) N (T ) = {0} and V = R(T ) + N (T ), by the denition of direct sum, V = R(T ) N (T ). Proof of (b). Let V be a nite-dimensional vector space, and let T : V V be linear. We wish to prove that V = R(T k ) N (T k ) for some k Z+ . By denition of direct sum, this is sucient to showing that R(T k ) N (T k ) = {0} and V = R(T k ) + N (T k ). In consideration of part (a), if we assume rank (T k ) = rank (T 2k ) then clearly R(T k ) N (T k ) = {0} following the same argument as we had shown. Similarly, in the same manner as the proof to Proposition (a), we have that R(T k ) + N (T k ) and V is nite-dimensional and dim(V ) = dim(R(T k ) + N (T k )) which means that V = R(T k ) + N (T k ) as required. So, since R(T k ) N (T k ) = {0} and dim(V ) = dim(R(T k ) + N (T k )), by denition of direct sum, V = R(T k ) N (T k ). 2.4.7. Let A be an n n matrix. (a) Suppose that A2 = O. Prove that A is not invertible. (b) Suppose that AB = O for some nonzero n n matrix B . Could A be invertible? Explain. Proof of (a). Let A be an n n matrix and suppose that A2 = O. We will prove by contradiction so suppose that A is invertible. Then, by denition of invertibility, there exists an n n matrix B such that AB = BA = I . We can rewrite such expression as B = A1 (note, equivalently, A = B 1 ). Then, since A2 = O, we have that A2 A1 = OA1 = O. However, since A is invertible and because A2 A1 = O, it 5

follows that A2 A1 = AAA1 = A(AA1 ) = AI = O which implies that A = O. This is a contradiction to the assumption that A is invertible (i.e., A = B 1 ). Thus, we conclude that A is not invertible. Proof of (b). We claim that A is not invertible. For the sake of contradiction, assume that A is invertible. Suppose that AB = O for some nonzero n n matrix B . Then, A1 AB = A1 O = O. But, because A1 AB = O and since A is invertible, A1 AB = (A1 A)B = IB = O which implies that B = O. However, this is a contradiction since we had that B is nonzero. Thus, A is not invertible. 2.4.9. Let A and B be n n matrices such that AB is invertible. Prove that A and B are invertible. Give an example to show that arbitrary matrices A and B need not be invertible if AB is invertible. Proof. Let A and B be n n matrices such that AB is invertible. Since AB is invertible, by denition of invertibility, there exists an n n matrix C such that ABC = CAB = In . Since ABC = A(BC ) = In , we have that A is invertible as BC is the multiplicative inverse of A by denition of invertibility (i.e., BC = A1 ). Similarly, since CAB = (CA)B = In , we have that B is invertible as CA is the multiplicative inverse of B by denition of invertibility (i.e., AB = B 1 ). Thus, we have that A and B are invertible. 1 0 1 0 0 Example: Let A = and B = 0 0 respectively. Clearly, A and B are not invertible as 0 0 1 0 1 1 0 they are not n n matrices (by denition of invertibility). However, their product is AB = which 0 1 is invertible as the identity is always is its own inverse. 2.4.10. Let A and B be n n matrices such that AB = In . (a) Use Exercise 9 to conclude that A and B are invertible. (b) Prove A = B 1 (and hence B = A1 ). (c) State and prove analogous results for linear transformations dened on nite-dimensional vector spaces. Proof of (a). Let A and B be n n matrices such that AB = In . Then, AB is invertible as the identity matrix is invertible. So since AB is invertible, by 2.4.9, we have that A is invertible and B is invertible. Proof of (b). From (a), we know that AB = In , A and B is invertible. Since A is invertible and AB = In , by denition of invertibility, B is the multiplicative inverse of A. That is, B = A1 . Then, it follows that B 1 = (A1 )1 = A by property of inverse. Proposition 2.4.10 (c). Let V and W be nite-dimensional vector spaces and let T : V W and U : W V be linear. If U T = IV then T U = IW . Proof of Proposition 2.4.10 (c). Let V and W be nite-dimensional vector spaces and let T : V W and U : W V be linear. Assume U T = IV . We wish to prove that T U = IW . For any v V , T U (T (v )) = T (U T (v )) = T (IV (v )) = T (v ). Since T (v ) W and v was arbitrary, it follows that T U = IW as required. Note that this suciently shows that T is invertible (U is the inverse of T ). 2.4.24. Let T : V Z be a linear transformation of a vector space V on to vector space Z . Dene the mapping : V /N (T ) Z by T (v + N (T )) = T (v ) T for any coset v + N (T ) in V /N (T ). (a) (b) (c) (d) Prove Prove Prove Prove that that that that is well-dened; that is, prove that if v + N (T ) = v + N (T ), then T (v ) = T (v ). T T is linear. is an isomorphism. T . the diagram shown in Figure 2.3 commutes; that is, prove that T = T

Proof of (a). Let T : V Z be a linear transformation of a vector space V on to vector space Z . Suppose (v + N (T )) = T (v ) and T (v + N (T )) = T (v ). Since v + N (T ) = v + N (T ) = v + N (T ). By denition, T v + N (T ), it follows that T (v ) = T (v ) as required. is linear which, by denition of linear transformation, is equivalent Proof of (b). We wish to prove that T (x + y ) = T (x) + T (y ) and T (cx) = cT (x). Choose to showing that for all x, y V /N (T ) and c F , T (a+b) = a, b V /N (T ) and c F . By denition of the mapping and the well-denedness of T from part (a), T (a + b) + N (T ) = (a + N (T )) + (b + N (T )) = T (a) + T (b) and T (ca) = (ca) + N (T ) = c(a + N (T )) = cT (a). is linear. Thus, we can conclude that T is linear and invertible. From part (b), we Proof of (c). By denition of isomorphism, we must show that T ) = have that T is linear. By Theorem 2.5, to show that T is invertible is equivalent to showing that dim(T dim(V ). By the Dimension Theorem, we know that dim(N (T )) + dim(R(T )) = dim((V )/N (T )). . Proof of (d). We wish to prove that T = T 2.5.10. Prove that if A and B are similar n n matrices, then tr(A) = tr(B ). Hint: Use Exercise 13 of Section 2.3. Proof. Suppose A and B are similar n n matrices. By denition, there exists an invertible matrix Q such that A = Q1 BQ. From Exercise 2.3.13, we had that tr(AB ) = tr(BA). It follows that tr(A) = tr(Q1 BQ) = ((Q1 B )Q) = tr((BQ1 )Q) = (B (Q1 Q)) = tr(B ) as required. 2.5.13. Let V be a nite-dimensional vector space over a eld F , and let = {x1 , x2 , ..., xn } be an ordered basis for V . Let Q be an n n invertible matrix with entries from F . Dene
n

xj =
n=1

Qij xi

1jn

and set = {x1 , x2 , ..., xn }. Prove that is a basis for V and hence that Q is the change of coordinate matrix changing -coordinates into -coordinates. 2.6.13. Let V be a nite-dimensional vector space of F . For every subset S of V , dene the annilator S 0 of S as S 0 = {f V : f (x) = 0 x S }. (a) (b) (c) (d) (e) Prove that S 0 is a subspace of V . If W is a subspace of V and x W , prove that there exists f W 0 such that f (x) = 0. Prove (S 0 )0 = span( (S )), where is dened as in Theorem 2.26. 0 0 For subspaces W1 and W2 , prove that W1 = W2 if and only if W1 = W2 . 0 0 0 For subspaces W1 and W2 , show that (W1 + W2 ) = W1 W2 .

Proof of (a). By Theorem 1.3, we wish to show that 0 S 0 and addition and scalar multiplication is closed for S 0 . Let f, g S 0 and a F . Clearly, f (0) = 0 S 0 . Next, by denition, (f + g )(x) = f (x)+ g (x) = 0+0 = 0 so S 0 is closed under addition. Lastly, by denition, af (x) = f (ax) = 0 so S 0 is closed under scalar multiplication. Thus, we have shown that S 0 is a subspace of V . Proof of (b). Suppose W is a subspace of V and x W . Let = {x1 , x2 , ..., xk } be a basis for W . Since W is a subspace of V , we can extend to V so 1 = {x1 , ..., xk , xk+1 , ..., xn }. Then, let (1 ) be the dual basis to 1 and dene (1 ) = {f1 , ..., fk , fk+1 , ..., fn }. Since we have that fi (xi ) = 0, W 0 {f1 , ..., fk }. Since xk+1 , ..., xn W , it follows that {fk+1 , ..., fn } W 0 . However, x W implies that for x = w + (a1 x1 + ... + ai xi ) where i > k , there exists an ao = 0. That is, fo (x) = ao = 0 as required. Proof of (c). () Pick s S such that (s) = s1 . Then, s1 (f ) = f (s) = 0 so (s) (S 0 )0 . Since and f is linear, it follows that

Proof of (d). Let W1 and W2 be subspaces.


0 0 () Suppose W1 = W2 . We wish to prove by contradiction so assume W1 = W2 . Then there exists 0 x W1 and x W2 . From part (b), we know that there exists f W1 such that f (x) = 0 which is a 0 0 0 contradiction since W1 = W2 and f (x) W2 . Thus, we have that W1 = W2 . 0 0 () Now suppose that W1 = W2 . Clearly by denition, since W1 = W2 , W1 = W2 . 0 0 Thus, we conclude that W1 = W2 if and only if W1 = W2 .

Proof of (e). () Let f (W1 + W2 )0 . Then, f (w1 ) = f (w2 ) = 0 for w1 W1 and w2 W2 . Then, we have 0 0 0 0 that f W1 and f W2 so (W1 + W2 )0 W1 W2 .
0 0 () Let f W1 W2 . Then, we have that f (W1 ) = 0 and f (W2 ) = 0 which implies that for w1 W1 and 0 0 w2 W2 , f (w1 + w2 ) = f (w1 ) + f (w2 ) = 0 + 0 = 0. That is, f (W1 + W2 )0 so W1 W2 ( W1 + W2 ) 0 . 0 0 0 0 Since we have that (W1 + W2 )0 W1 W2 and W1 W2 (W1 + W2 )0 , we can conclude that (W1 + W2 )0 = 0 0 W1 W2 .

2.6.14. Prove that if W is a subspace of V , then dim(W ) + dim(W 0 ) = dim(V ). Hint: Extend an ordered basis {x1 , x2 , ..., xk } of W to an ordered basis = {x1 , x2 , ..., xn } of V . Let = {f1 , f2 , ..., fn }. Prove that {fk+1 , fk+2 , ..., fn } is a basis for W 0 . Proof. Suppose W is a subspace of V . Let W = {x1 , x2 , ..., xk } be an ordered basis of W and extend it onto an ordered basis = {x1 , x2 , ..., xn } of V . Let = {f1 , f2 , ..., fn }. We wish to show that = {fk+1 , fk+2 , ..., fn } is a basis for W 0 which is equivalent to showing that dim(W ) + dim(W 0 ) = dim(V ). Clearly, W 0 and it must be linearly independent since it is a subset of a basis. By denition of basis, we need to show that span W 0 . We will prove by contradiction so assume that does not span W 0 . Since W 0 V , we have that a1 f1 + ... + an fn W 0 where a1 , ..., an are not all zero. That is, there exists an ao such that ao = 0. This implies that f (vo ) = ao = 0 for vo W , a contradiction. Thus, span W 0 . Since is linearly independent and since it generates W 0 , we have that is a basis for W 0 as required. 2.6.15. Suppose that W is a nite-dimensional vector space and that T : V W is linear. Prove that N (T t ) = (R(T ))0 . Proof. Suppose that W is a nite-dimensional vector space and that T : V W is linear. () Let f N (T t ). By denition of nullspace, we have that T t (f ) = 0. Then, for all v V , f (T (v )) = 0. This implies that f (R(T )) = 0 which, by denition of annilator, means that f (R(T ))0 . Thus, N (T t ) (R(T ))0 . () Now let f (R(T ))0 . By denition of annilator, we have that f (R(T )) = 0. By denition of range, this means that for all v V , we have that f (T (v )) = 0. Since f (T (v )) = 0, we have that T t (f ) = 0 so f N (T t ) by denition of nullspace. Thus, (R(T ))0 N (T t ). Since N (T t ) (R(T ))0 and (R(T ))0 N (T t ), we have that N (T t ) = (R(T ))0 . 2.6.16. Use Exercises 14 and 15 to deduce that rank (LAt ) = rank (LA ) for any A Mmn (F ). Proof. We wish to show that rank (LAt ) = rank (LA ) for any A Mmn (F ). Note, LA : V W and LAt : W V . Also, by denition of dual space, we know that dim(V ) = dim(V ) and dim(W ) = dim(W ). So, from denition of rank and by 2.6.14 (Homework Set 5 ) and 2.6.15 we have that

rank (LAt ) = dim(R(LAt )) = dim(W ) dim(W ) dim(R(LAt )) = dim(W ) dim(N (LAt )) = dim(W ) dim((R(LA ))0 ) = dim(R(LA )) = rank (LA ) as required.

Partial Solutions for Linear Algebra by Friedberg et al. Chapter 3


John K. Nguyen December 7, 2011
3.2.14. Let T, U : V W be linear transformations. (a) Prove that R(T + U ) R(T ) + R(U ) (b) Prove that if W is nite-dimensional, then rank (T + U ) rank (T ) + rank (U ). (c) Deduce from (b) that rank (A + B ) rank (A) + rank (B ) for any m n matrices A and B . Proof of (a). Let T, U : V W be linear transformations. We wish to prove that R(T + U ) R(T ) + R(U ) so assume that x R(T + U ). Then, by denition of range, there exists v V such that (T + U )(x) = v . By denition of linear transformation, we have that T (x) + U (x) = v which implies that x R(T ) + R(U ) according to the denition of range. Thus, R(T + U ) R(T ) + R(U ). Proof of (b). Assume W is nite-dimensional. By Theorem 2.1, we know that R(T ) and R(U ) are subspaces of W and, as such, they are both nite-dimensional. From (a), we know that R(T + U ) R(T ) + R(U ) so we have dim(R(T + U )) dim(R(T ) + R(U )) dim(R(T )) + dim(R(U )). By denition of rank, we have that rank (T + U ) rank (T ) + rank (U ). Proof of (c). Suppose A and B are both m n matrices. By denition of left-multiplication transformation, we have that LA : F m F n and LB : F m F n . In consideration of (b) and Theorem 2.15, we have that rank (LA+B ) = rank (LA + LB ) rank (LA ) + rank (LB ) which implies that rank (A + B ) rank (A) + rank (B ) as required. 3.2.17. Prove that if B is a 3 1 matrix and C is a 1 3 matrix, then the 3 3 matrix BC has rank at most 1. Conversely, show that if A is any 3 3 matrix having rank 1, then there exists a 3 1 matrix B and a 1 3 matrix C such that A = BC . Proof. Suppose B is an 3 1 matrix and C is an 1 3 matrix. By Theorem 3.5, we know that the rank of any matrix equals the maximum number of its linearly independent columns. Thus, clearly rank (B ) 1 since B is 3 1. By denition of matrix multiplication, we have that BC is an 3 3 matrix so it is dened. Then, from Theorem 3.7, we have that rank (BC ) rank (B ). Since we know that rank (B ) 1, it follows that rank (BC ) rank (B ) 1 which implies that rank (BC ) 1. To show the converse, suppose A is any 3 3 matrix having rank 1. By denition, we have that LA : F 3 F 3 . Let be the standard ordered basis for F 3 and assume f : F 1 . Then, by the Freeness Theorem, there exists a unique linear transformation LB : F 3 F 1 extending f . Now suppose is a standard ordered basis for F 1 and assume g : F 3 . Also by the Freeness Theorem, there exists a unique linear transformation LC : F 1 F 3 extending g . So, since LB : F 3 F 1 , we have that B is a 3 1 matrix. Similarly, we know that C is a 1 3 matrix. It follows that LB LC : F 3 F 3 which implies that BC is 3 3 (we also know this by denition of matrix multiplication). Thus, A = BC as required. 3.2.19. Let A be an m n matrix with rank m and B be an n p matrix with rank n. Determine the rank of AB . Justify your answer. Proof. We claim that rank (AB ) = m.

Let A be an m n matrix with rank m and B be an n p matrix with rank n. By denition of matrix multiplication, we know that AB is an m p matrix. By denition, we have that LA : F m F n , LB : F n F p and LAB : F m F p respectively. Observe that since LB : F p F n is onto (recall Theorem 2.15 ), we have R(AB ) = R(LA LB ) = LA LB (F p ) = LA (LB (F p )) = LA (F n ) = R(LA ). Therefore, rank (AB ) = dim(R(LA LB )) = dim(R(LA )) = rank (A). And since rank (A) = m, we have that rank (AB ) = m.

Partial Solutions for Linear Algebra by Friedberg et al. Chapter 4


John K. Nguyen December 7, 2011
4.1.11. Let : M22 (F ) F be a function with the following three properties. (i) is a linear function of each row of the matrix when the other row is held xed. (ii) If the two rows of A M2 (F ) are identical, then (A) = 0. (iii) If I is the 2 2 identity matrix, then (I ) = 1. Prove that (A) = det(A) for all A M22 (F ). Proof. Let A M22 (F ) and dened it as above, notice that 0 1 1 0 1 1 1 1 = 1 0 0 1 a c b d where a, b, c, d are scalars. First, by the properties 0 1 1 0 = 1+ 0 1 1 0 = 0 which implies that

= 1. Now, in knowing this and from the properties above, we have the following a c 1 c 1 1 b d 0 d 0 0 + b + ad 0 c 1 d 1 0 0 1 + bc 0 1 1 0 + bd 0 0 1 1

(A) = = a = ac

= ac(0) + ad(1) + bc(1) + bd(0) = ad bc = det = det(A). Since A was arbitrary, we have shown that (A) = det(A). 4.2.25. Prove that det(kA) = k n det(A) for any A Mnn (F ). Proof. Choose A Mnn (F ). Then, by Theorem 4.8, det(kA) = det(kIn A) = det(kIn )det(A) = k n det(In )det(A) = k n 1det(A) = k n det(A). Since A was arbitrary, we have that det(kA) = k n det(A) for any A Mnn (F ) as required. a c b d

4.3.10. A matrix M Mnn (F ) is called nilpotent if, for some positive integer k , M k = 0, where 0 is the n n zero matrix. Prove that if M is nilpotent, then det(M ) = 0. Proof. Suppose M Mnn (F ) such that it is nilpotent. Then, by denition, there exists some k Z such that M k = 0, where 0 is the n n zero matrix. This implies that det(M k ) = 0. By Theorem 4.7, we know that det(AB ) = det(A)det(B ) for any A, B Mnn (F ). So, by induction, we know that det(M k ) = (det(M ))k for all k Z. Since the determinant of the zero matrix is zero, (det(M ))k = 0 which implies that det(M ) = 0 as required. 4.3.11. A matrix M Mnn (F ) is called skew-symmetric if M t = M . Prove that if M is a skew-symmetric and n is odd, then M is not invertible. What happens if n is even? Proof. Choose M Mnn (F ) such that it is skew-symmetric. Then, by Theorem 4.8 and since M t = M , det(M ) = det(M t ) = det(M ) = (1)n det(M ). Since we know that n is odd, we have that det(M ) = det(M ). Rearranging terms, we have that 2det(M ) = 0 which implies that det(M ) = 0 so, by the corollary on page 223, M is not invertible. If n is even, then we would have det(M ) = det(M ) which would not imply anything. 4.3.13. For M Mnn (C ), let M be the matrix such that (M )ij = Mij for all i, j , where Mij is the complex conjugate of Mij . (a) Prove that det(M ) = det(M ). (b) A matrix Q Mnn (C ) is called unitary if QQ = I , where Q = Qt . Prove that if Q is a unitary matrix, then |det(Q)| = 1. Proof of (a). Let M Mnn (C ) and suppose M be the matrix such that (M )ij = Mij for all i, j , where Mij is the complex conjugate of Mij .

Proof of (b). Let Q Mnn (C ) be a unitary matrix. Then, by denition, QQ = QQt = I . Then, from Theorem 4.8 and part (a), we have that det(I ) = det(QQt ) = det(Q)det(Qt ) = det(Q)det(Qt ) = det(Q)det(Q) = |det(Q)|2 . Since det(I ) = 1, we have that |det(Q)|2 = 1 which can further be reduced to |det(Q)| = 1 as required. 4.3.21. Prove that if M Mnn (F ) can be written in the form M= A O B C

where A and C are square matrices, then det(M ) = det(A) det(C ). Proof. Let A be an k k matrix, B be a k t matrix and C be a t t matrix. We wish to prove by induction. For our base case, suppose k = 1. Then, expanding down the rst column, 11 ) + 0 + ... + 0 = a det(C ) = det(A)det(C ). det(M ) = a det(M So, this holds true for k = 1. Now suppose that A is an (k 1) (k 1) matrix and B is an (k 1) t matrix and C is a t t matrix and that A B M= . O C Now, taking the cofactor expansion along the rst column, 11 ) ... ak1 det(M det(M ) = a11 det(M k1 ) + 0 + ... + 0 A11 B1 A k 1 Bk = a11 det ... det O C O C = a11 det(A11 )det(C ) ... ak1 det(Ak1 )det(C ) = (a11 det(A 11 )) ... ak1 det(Ak1 )det(C ) = det(A)det(C ).

4.3.22. Let T : Pn (F ) F n+1 be the linear transformation dened in Exercise 22 of Section 2.4 by T (f ) = (f (c0 ), f (c1 ), ..., f (cn )), where c0 , c1 , ..., cn are distinct scalars in an innite eld F. Let be the standard ordered basis for Pn (F ) and be the standard ordered basis for F n+1 . (a) Show that M = [T ] has the form

(c) Prove that det(M ) =


0i<j n

(cj ci ),

the product of all the terms of the of the form cj ci for 0 i < j n.

Proof of (a). Suppose T : Pn (F ) F n+1 is the linear transformation dened in Exercise 22 of Section 2.4 by T (f ) = (f (c0 ), f (c1 ), ..., f (cn )), where c0 , c1 , ..., cn are distinct scalars in an innite eld F. Let be the standard ordered basis for Pn (F ) and be the standard ordered basis for F n+1 .

Proof of (c). We will prove that det(M ) =


0i<j n

(cj ci ),

the product of all the terms of the of the form cj ci for 0 i < j n.

Partial Solutions for Linear Algebra by Friedberg et al. Chapter 5


John K. Nguyen December 7, 2011
5.2.11. Let A be an n n matrix that is similar to an upper triangular matrix and has the distinct eigenvalues 1 , 2 , ..., k with corresponding multiplicities m1 , m2 , ..., mk . Prove the following statements.
k

(a) tra(A) =
i=1

mi i .

(b) det(A) = (1 )m1 (2 )m2 ...(k )mk . Proof of (a). Since A be an n n matrix that is similar to an upper triangular matrix, say B, that has the distinct eigenvalues 1 , 2 , ..., k with corresponding multiplicities m1 , m2 , ..., mk . By denition of similar, there exists an invertible matrix Q such that A = Q1 BQ. Then, in consideration of Exercise 2.3.13,
k

tr(A) = tr(Q

BQ) = tr(Q

QB ) = tr(B ) =
i=1

mi i .

Proof of (b). Since A be an n n matrix that is similar to an upper triangular matrix, say B, that has the distinct eigenvalues 1 , 2 , ..., k with corresponding multiplicities m1 , m2 , ..., mk . By denition of similar, there exists an invertible matrix Q such that A = Q1 BQ. Recall Theorem 4.7 which states that for any A, B M22 (F ), det(AB ) = det(A)det(B ). In consideration of this theorem and the corollary on page 223, we have that det(A) = det(Q1 BQ) = det(Q1 )det(B )det(Q) = det1 (Q) det(B )det(Q) = det(B ) = (1 )m1 (2 )m2 ...(k )mk as required. 5.2.12. Let T be an invertible linear operator on a nite-dimensional vector space V . (a) Recall that for any eigenvalue of T , 1 is an eigenvalue of T 1 . Prove that the eigenspace of T corresponding to is the same as the eigenspace of T 1 corresponding to 1 . (b) Prove that if T is diagonalizable, then T 1 is diagonalizable. Proof of (a). Pick v E . Then, by denition, T (v ) = v . Taking the inverse of both sides, we get T 1 T (v ) = T 1 (v ) which means v = T 1 (v ). Then, by denition, v E1 so we have that the eigenspace of T corresponding to is the same as the eigenspace of T 1 corresponding to 1 . Proof of (b). Suppose T is diagonalizable. Then T has n linearly independent eigenvectors. From part (a) we know that T 1 also has the same n eigenvectors. Thus, T 1 is diagonalizable. 5.2.13. Let A Mnn (F ). Recall from Exercise 14 of Section 5.1 that A and At have the same characteristic polynomial and hence share the same eigenvalues with the same multiplicities. For any eigenvalue of A and At , let E and E denote the corresponding eigenspaces for A and At , respectively. (a) Show by way of example that for a given common eigenvalue, these two eigenspaces need not be the same. (b) Prove that for any eigenvalue , dim(E ) = dim(E ). (c) Prove that if A is diagonalizable, then At is also diagonalizable.

Example for (a). Dene A =


t EA = EA .

1 3

2 4

. Now, notice that A(1, 0) = (1, 3) but At (1, 0) = (1, 2). Thus,

Proof of (b). Suppose dim(F ) = n. Then, by denition, we know that dim(E ) = n rank (A I ). Then, taking the transpose (recall that rank (A) = rank (At ) by a previous exercise), we have that dim(E ) = n rank ((A I )t ) = n rank (At I ) = dim(E ) as required. Proof of (c). Suppose A is diagonalizable. Then, there exists an invertible matrix Q such that B = Q1 AQ is a diagonal matrix (recall that a square matrix is diagonalizable if it is similar to a diagonal matrix according to Section 5.1 ). Now, taking the transpose of both sides yields B t = (Q1 AQ)t = Qt At (Q1 )t . Clear,y B t is diagonal so by denition, we have that At is diagonalizable. 5.2.18a. Prove that if T and U are simultaneously diagonalizable operators, then T and U commute (i.e., U T = T U ). Proof. Suppose T and U are simultaneously diagonalizable operators. Then, by denition, there exists an ordered basis = {v1 , v2 , ..., vn } such that T (vi ) = vi and U (vi ) = vi where i = 1, 2, ..., n. It follows that T U (vi ) = T (U (vi )) = T (vi ) = T (vi ) = vi = vi = U (vi ) = U (vi ) = U (T (vi )) = U T (vi ). So, we can conclude that U T = T U as required. 5.4.13. Let T be a linear operator on a vector space V , let v be a nonzero vector in V , and let W be the T-cyclic subspace of V generated by v . For any w V , prove that w W if and only if there exists a polynomial g (t) such that w = g (T )(v ). Proof. () Suppose w W and assume dim(W ) = n. Let = {v, T v, ..., T n1 v } be an ordered basis for W . Then, by denition, w = a0 v + a1 T v + ... + an1 T n1 v for some scalars a0 , a1 , ...an1 (that is, w is a linear combination of the elements of ). Let g (t) = a0 + a1 t + ... + an1 tn1 . Then, g (t) is a polynomial of t of degree less than or equal to n 1. This also means that w = g (T )v . () Suppose there exists a polynomial g (t) such that w = g (T )v . Then, g (t) = a0 + a1 t + ... + an tn and w = a0 v + a1 T v + ... + an T n v for some scalars a0 , a1 , ..., an . Since v, T v, ..., T n v W , w W . 5.4.16. Let T be a linear operator on a nite-dimensional vector space V . (a) Prove that if the characteristic polynomial of T splits, then so does the characteristic polynomial of the restriction of T to any T-invariant subspace of V . (b) Deduce that if the characteristic polynomial of T splits, then any nontrivial T-invariant subspace of V contains an eigenvector of T . Proof of (a). Suppose the characteristic polynomial of T , say f , splits. Let W be a T invariant subspace of V and g is the characteristic polynomial of TW . Then, g divides f so there exists a polynomial r such that f = gr. Suppose that f has n degrees. Note that the amount of zeros that g have is less than or equal to degree of g and analogously for r. But, it follows that n = deg (g ) = deg (f ). So, g has deg (g ) zeros, h has deg (h) zeros. Thus, we can factor g to deg (g ) factors which means that the characteristic polynomial of the restriction of T to a T-invariant subspace splits. Proof of (b). Suppose W is a nontrivial T-invariant subspace of V . Let f be the characteristic polynomial of TW . By part (a), we know that f splits. Pick to be the root of f . Then, f () = det(TW I ) = 0 . But this means that TW I is not invertible so there exists a nonzero w W such that (T I )(w) = 0 which implies that w is an eigenvector of T . Since w is arbitrary, we have shown that if the characteristic polynomial of T splits, then any nontrivial T-invariant subspace of V contains an eigenvector of T . 5.4.20. Let T be a linear operator on a vector space V , and suppose that V is a T-cyclic subspace of itself. Prove that if U is a linear operator on V , then U T = T U if and only if U = g (T ) for some polynomial g (t). Hint: Suppose that V is generated by v . Choose g (t) according to Exercise 13 so that g (T )(v ) = U (v ).

Proof. () Suppose U T = T U . Then, since V = span(v, T (v ), T 2 (v ), ...), U (v ) = a0 + a1 T (v ) + ... + an T n (v ) for some scalars a0 , a1 , ..., an . So, U (v ) = g (T )(v ) where g (v ) = a0 + a1 x + ... + an xn . Suppose x V . Then, x = b0 + b1 T (v ) + ... + bm T m (v ) for some scalars b0 , b1 , ..., bm . It follows that U (x) = U (b0 + b1 T (v ) + ... + bm T m (v )) = b0 U (T 0 (v )) + b1 U (T (v )) + ... + bm U (T m (v )) = b0 T 0 (U (v )) + b1 T (U (v )) + ... + bm T m (U (v )) = b0 T 0 (g (T )(v )) + b1 T (g (T )(v )) + ... + bm T m (g (T )(v )) = b0 g (T )(T 0 (v )) + b1 g (T )(T (v )) + ... + bm g (T )(T m (v )) = g (T )(b0 T 0 (v ) + b1 T 1 (v ) + ... + bm T m (v )) = g (T )(x). Thus, U = g (T ) for some polynomial g . () Suppose U = g (T ) for some polynomial g . Dene g (x) = a0 + a1 x + ... + an xn . Then, U T (x) = a0 T 0 (T (x)) + a1 T (T (x)) + ... + an T n (T (x)) = a0 T (T 0 (x)) + a1 T (T (x)) + ... + an T (T n (x)) = T (a0 T 0 (x) + a1 T (x) + ... + an T n (x)) = T U (x). Therefore, we have that U T = T U . We have shown that if U is a linear operator on V , then U T = T U if and only if U = g (T ) for some polynomial g (t).

Partial Solutions for Linear Algebra by Friedberg et al. Chapter 6


John K. Nguyen December 7, 2011
6.1.18. Let V be a vector space over F , where F = R or F = C , and let W be an inner product space over F with product , . If T : V W is linear, prove that x, y = T (x), T (y ) denes an inner product on V if and only if T is one-to-one. Proof. () Suppose x, y = T (x), T (y ) denes an inner product on V . Let T (x) = T (y ) which, since T is linear, implies T (x) T (y ) = T (x y ) = 0. From our denition above, we have that x y, x y = T (x y ), T (x y ) = T (x), T (x y ) T (y ), T (x y ) = 0. By denition (particularly part (d) on page 330) it clearly follows that x y = 0 which means x = y so T is one-to-one. () Assume that T is one-to-one. That is, T (x) = T (y ) implies x = y . Then, x + z, y = T (x + z ), T (y ) = T (x), T (y ) + T (z ), T (y ) . Since W is an inner product space, we have that x + z, y = x, y + z, y so it follows immediately that T (x), T (y ) = x, y . We have proven that x, y = T (x), T (y ) denes an inner product on V if and only if T is one-to-one. 6.1.12. Let {v1 , v2 , ..., vk } be an orthogonal set in V , and let a1 , a2 , ..., ak be scalars. Prove that
k 2 k

a i vi
i=1

=
i=1

|ai |2 ||vi ||2 .

Proof. We apply the denition of inner product space and Theorem 6.2 to get the following
k 2 k k k k k

ai vi
i=1

=
j =1

aj vj ,
i=1

ai v i

=
j =1 i=1

aj a i < vj , vi >=
i=1

|ai |2 ||vi ||2 .

as required. 6.2.13. Let V be an inner product space, S and S0 be subsets of V , and W be a nite-dimensional subspace of V . Prove the following results. (a) S0 S implies that S S0 . (b) S (S ) ; so span(S ) (S ) . (c) W = (W ) . Hint: Use Exercise 6. (d) V = W W . (See the exercises of Section 1.3) Proof of (a). Suppose S0 S . Let x S . Then, by denition, for all y S , < x, y >= 0 which implies that y S0 . But S0 S , so x S0 . Thus, S S0 as required. Proof of (b). Let x S . Then, < x, y >= 0 for all y S . But this also means that x (S ) . So, S (S ) . Proof of (c). () By part (b), we have that W (W ) . () We will prove the contrapositive so assume that x W . Then, by Exercise 6.2.6, there exists y W 1

such that < x, y >= 0. This implies that x (W ) . Thus, we have shown that W = (W ) . Proof of (d). By denition of direct sum, we wish to show that W W = {0} and V = W + W . Since the only vector that is orthogonal to itself is the zero vector, we have that W W = {0}. Let v V . By Theorem 6.6, there exist unique vector u W and z W such that v = u + z so we have that V = W + W . We have shown that V = W W . 6.2.14. Let W1 and W2 be subspaces of a nite-dimensional inner product space. Prove that (W1 + W2 ) = W1 W2 and (W1 W2 ) = W1 + W2 . Hint for the second equation: Apply Exercise 13(c) to the rst equation.
Proof of (a). () Let x (W1 + W2 ) . By denition, it follows immediately that x W1 W2 . So, (W1 + W2 ) W1 W2 . () Let x W1 W2 . Then, x W1 and x W2 . By linearity, it follows that x (W1 + W2 ) . So, (W1 + W2 ) W1 W2 . Thus, since (W1 + W2 ) W1 W2 and (W1 + W2 ) W1 W2 , (W1 + W2 ) = W1 W2 . Proof of (b). () Let x (W1 W2 ) . Then, since W1 W1 + W2 and W2 W1 + W2 by a previous exercise, it follows that x W1 + W2 so (W1 W2 ) W1 + W2 (since x is perpendicular to vectors that is in both W1 and W2 ). () Let x W1 + W2 . Then, by denition, x = w1 + w2 where w1 W1 and w2 W2 . But, W1 W1 + W2 and W2 W1 + W2 so, in consideration of Exercise 6.2.13c, (W1 W2 ) W1 + W2 which means that x (W1 W2 ) (i.e., w1 and w2 is perpendicular to the intersection of W1 and W2 ). Therefore, (W1 W2 ) W1 + W2 . In conclusion, since (W1 W2 ) W1 + W2 and (W1 W2 ) W1 + W2 , we have that (W1 W2 ) = W1 + W2 .

6.2.22. Let V = C ([0, 1]) with the inner product < f, g >= the linearly independent set {t, t}. (a) Find an orthonormal basis for W .

1 0

f (t)g (t)dt. Let W be the subspace spanned by

(b) Let h(t) = t2 . Use the orthonormal basis obtained in (a) to obtain the best (closest) approximation of h in W . 6.3.6. Let T be a linear operator on an inner product space V . Let U1 = T + T and U2 = T T . Prove that U1 = U1 and U2 = U2 . Proof. Let T be a linear operator on an inner product space V and suppose that U1 = T + T and U2 = T T . We rst prove that U1 = U1 . By our assumption and Theorem 6.11,
U1 = (T + T ) = T + (T ) = T + T = U1 . In a similar argument, we will show that U2 = U2 . U2 = (T T ) = T (T ) = T T = U2 . Thus, we have that U1 = U1 and U2 = U2 as required.

6.3.8. Let V be a nite-dimensional inner product space, and let T be a linear operator on V . Prove that if T is invertible, then T is invertible and (T )1 = (T 1 ) . Proof. Let V be a nite-dimensional inner product space, and let T be a linear operator on V . Choose x, y V . Suppose T is invertible. Then, T (T 1 ) (x), y = (T 1 ) (x), T (y ) = x, T 1 T (y ) = x, y which implies that T (T 1 ) = I . Therefore, (T )1 = (T 1 ) . 6.3.9. Prove that if V = W W and T is the projection on W along W , then T = T . Hint: Recall that N (T ) = W . Proof. Suppose that V = W W and T is the projection on W along W . Since V = W W , we have that V = W + W . Let v1 , v2 V . Then, there exists w1 , w2 W and w 1 , w 2 W such that v1 = w1 + w 1 and v2 = w2 + w 2 . Now, in consideration that T is the projection on W along W , v1 , T (v2 ) = w1 + w 1 , T (w2 + w 2 ) = w1 + w 1 , w2 = w1 , w2 . Similarly, T (v1 ), v2 = T (w1 + w 1 ), w2 + w 2 = w1 , w2 + w 2 = w1 , w2 . So, it follows that v1 , T (v2 ) = T (v1 ), v2 which means v1 , T (v2 ) = v1 , T (v2 ) . This implies that T = T 6.3.11. For a linear operator T on an inner product space V , prove that T T = T0 implies T = T0 . Is the same result true if we assume that T T = T0 . Proof. Let T be a linear operator on an inner product space V . Suppose T T = T0 . Pick x V . Then, T T (x), x = T0 (x), x = 0, x = 0 by Theorem 6.1(c). But, we also have that T T (x), x = T (x), T (x) so T (x), T (x) = ||T (x)||2 = 0. It follows that T (x) = 0 which implies T = T0 . We claim that the same result is true if we assume that T T = T0 .

Proof. Let T be a linear operator on an inner product space V . Suppose T T = T0 . Pick x V . Then, T T (x), x = T0 (x), x = 0, x = 0. But T T (x), x = T (x), T (x) which, analogously to the argument above, we have that T (x) = 0 so T = T0 . Then, (T ) = T0 so T = T0 (note that the adjoint of the zero operator is the zero operator). 6.3.12. Let V be an inner product space, and let T be a linear operator on V . Prove the following results: (a) R(T ) = N (T ) (b) If V is nite-dimensional, then R(T ) = N (T ) . Hint: Use Exercise 13(c) of Section 6.2. Proof of (a). By denition, x R(T ) if and only if x, T (y ) = T (x), y = 0 for all y V . By Theorem 6.1, this is true if and only if T (x) = 0 which means x N (T ). Thus, x R(T ) if and only if x N (T ) so we have that R(T ) = N (T ). Proof of (b). Suppose V is nite-dimmensional. From part (a), we know that R(T ) = N (T ). It follows that (R(T ) ) = N (T ) . In consideration of 6.2.13c, we have that (R(T ) ) = R(T ). Thus, since (R(T ) ) = N (T ) and (R(T ) ) = R(T ), we have that R(T ) = N (T ) .

You might also like